LSAT and Law School Admissions Forum

Get expert LSAT preparation and law school admissions advice from PowerScore Test Preparation.

User avatar
 Dave Killoran
PowerScore Staff
  • PowerScore Staff
  • Posts: 5850
  • Joined: Mar 25, 2011
|
#27286
Complete Question Explanation
(The complete setup for this game can be found here: lsat/viewtopic.php?t=1171)

The correct answer choice is (D)

If exactly four of the cars have L, those cars must be T, W, X, and Z, and if exactly four of the cars have P, those cars must be V, W, X, and Z. This information leads to the following setup:
Oct 01_M12_game#2_L9_explanations_game#1_#11_diagram_1.png
Because W and Z have the exact same two options, answer choice (D) cannot be true and is the correct answer.
User avatar
 cici5579
  • Posts: 2
  • Joined: Nov 03, 2021
|
#93143
Hi! I was wondering why answer choice (E) was wrong. Based on the diagram, it indicates that Y and Z have no options in common either. However, if we use the same diagram from question #10, I see how (E) is wrong. I appreciate any help you can provide.
 Adam Tyson
PowerScore Staff
  • PowerScore Staff
  • Posts: 5153
  • Joined: Apr 14, 2011
|
#93158
This is an EXCEPT question, cici5579, so the four wrong answers are all ones that must be true, and the correct answer need not be true. Answer E must be true - Y and Z cannot have anything in common in this local scenario as shown by the diagram - so it is a wrong answer! Don't refer back to the diagram from question 10, because that diagram does not comply with this question.

Get the most out of your LSAT Prep Plus subscription.

Analyze and track your performance with our Testing and Analytics Package.